OlimpíadasNúmeros primos

Aqui devem ser postados problemas Olímpicos. Informe a olimpíada e o ano no título do tópico. Exemplo: (OBM - 2008).

Moderador: [ Moderadores TTB ]

Avatar do usuário
Autor do Tópico
majik
sênior
Mensagens: 30
Registrado em: Sáb 22 Mar, 2014 15:56
Última visita: 19-07-15
Abr 2014 13 14:21

Números primos

Mensagem não lida por majik »

Encontre todos os primos [tex3]p[/tex3] , para os quais o quociente.

[tex3]\frac{2^{p-1}-1}{p}[/tex3]
é um quadrado perfeito.

Última edição: majik (Dom 13 Abr, 2014 14:21). Total de 1 vez.



Auto Excluído (ID:12031)
6 - Doutor
Última visita: 31-12-69
Jun 2014 19 22:36

Re: Números primos

Mensagem não lida por Auto Excluído (ID:12031) »

Cara, não consegui resolver ainda mas vou deixar aqui algumas observações que eu acho que podem ajudar a resolver esse problema:

Se p=2 teremos:

\frac{2^{1}-1}{2} =\frac{1}{2}
então p=2 não nos convém.

Pelo pequeno teorema de fermat se mdc(a,p)=1 e p é primo então:

a^{p-1}\equiv1\, mod(p)

Se p\neq 2 então o pequeno teorema de fermat garante que
2^{p-1} -1 é múltiplo de p

o problema pede que:
2^{p-1} -1=p\cdot n^{2}

como p é ímpar podemos dizer que 2^{p-1} é um quadrado perfeito. Pois o expoente é par. Mais do que isso 2^{p-1} não é divisível por 3. Pois é uma potência de 2.

Lema: Todo quadrado perfeito m^2 quando dividido por 3 deixa restos:
1 - se m não for múltiplo de 3
0 - se m é múltiplo de 3

Prova: ou m = 3k pra k inteiro positivo então m^2=9k^2=3\cdot(3k^2)
ou m = 3k+1 então m^2 = 9k^2+6k+1=3(3k^2+2k)+1
ou m=3k+2 então m^2=9k^2+12k+4=9k^2+12k+3+1=3(3k^2+4k+1)+1
cqd

então temos que 2^{p-1} \equiv 1\, mod(3) \rightarrow  2^{p-1} - 1 \equiv 0\, mod(3) então devemos ter que:
p \cdot n^2 \equiv 0\, mod(3)
excetuando-se o caso p=3 que satisfaz a condição do problema pois:
\frac{2^{2}-1}{3}=1
devemos ter então n=3k e é fácil ver que n deve ser ímpar, pois 2^{p-1}-1 é ímpar e p também é. Então n=3(2m-1).
Com esta descoberta podemos inferir agora que se p>3 então 2^{p-1}-1 \equiv 0 \, mod(9)
observemos que:
2^0 \equiv 1\,mod(9)
2^1 \equiv 2\,mod(9)
2^2 \equiv 4\,mod(9)
2^3 \equiv 8\,mod(9)
2^4 \equiv 2\cdot8\,mod(9)\equiv 16\, mod(9) \equiv 7\, mod(9)
2^5 \equiv 2\cdot7 \,mod(9)\equiv 5 \, mod(9)
2^6 \equiv 2\cdot5\,mod(9)\equiv 10\,mod(9) \equiv 1\,mod(9)
e a partir dai o padrão se repete.
Então 2^{n}-1 só é divisível por 9 se n=6k pra algum k inteiro não-negativo, então temos que p-1=6k \rightarrow p=6k+1 (*)

teremos:
2^{p-1}-1=p \cdot 9\, \cdot (2m-1)^2

Lema: todo quadrado perfeito ímpar deixa resto 1 quando dividido por 4. Todo quadrado perfeito par é divisível por 4.
Prova: Seja m=2k-1 então m^2=4k^2-4k+1=4k(k-1)+1

pra os primos p com p>3 temos que 2^{p-1} é divisível por 4 então:
2^{p-1} -1 \equiv 3 \, mod(4)
do lado direito:
9\, \cdot p \cdot (2m-1)^2 \equiv p\,mod(4)
então
p \equiv 3 \, mod(4) \rightarrow  p =4x +3(**)

(*) e (**): 6k+1=4x+3 \rightarrow 3k=2x+1\rightarrow x=\frac{3k-1}{2} = k-1+\frac{k+1}{2}
k+1=2y \rightarrow  k =2y-1 \rightarrow x = 3y-2
vou só mostrar que x deve ser ímpar e deixar indicada a PA em que p se encontra. Pelo teorema de Dirichlet para progressões aritméticas, temos que existem infinitos primos nessa sequência, então eu não resolvi o problema, mas o restringi um pouco. Espero que isso sirva pra alguém.

Lema: todo quadrado perfeito ímpar deixa resto 1 quando dividido por 8.
Prova: Seja m=2k-1 então m^2=4k^2-4k+1=4k(k-1)+1
é fácil ver que k(k-1) é par.
pra os primos p com p>3 temos que 2^{p-1} é divisível por 8 então:
2^{p-1} -1 \equiv 7 \, mod(8)
do lado direito:
9\, \cdot p \cdot (2m-1)^2 \equiv p\,mod(8)
então
p \equiv 7 \, mod(8) \rightarrow  p =8z +7 = 4x + 3\rightarrow  x=2z+1cqd
como
x=3y-2 então y=2i-1
p=4(3(2i-1)-2)+3 =4(6i-5)+3=24i-17
alguns candidatos a solução são: 7,31,79,103,127,...

Última edição: Auto Excluído (ID:12031) (Qui 19 Jun, 2014 22:36). Total de 1 vez.



Auto Excluído (ID:12031)
6 - Doutor
Última visita: 31-12-69
Jan 2015 09 11:11

Re: Números primos

Mensagem não lida por Auto Excluído (ID:12031) »

2^{p-1} = 2^{24i-18} = 2^{24k+6} = 2^{6(4k+1)}
2^6 \equiv 8\cdot8\mod(7)\equiv1 \mod(7)
2^{24} \equiv (2^6)^4\mod(7) \equiv 1 \mod(7)

logo para todo primo da forma 24i-17 2^{p-1} - 1 \equiv 0\mod(7)
excetuando-se o caso p=7 isso implica que n = 9 \cdot 49 \cdot (2m-1)^2

2^{24i-18} só vai ser 1 \mod(49) quando i = 6 + 7k
p-1 = 24(6+7k) - 18 = 168k + 126
p = 168k + 127
Última edição: Auto Excluído (ID:12031) (Sex 09 Jan, 2015 11:11). Total de 1 vez.



Avatar do usuário
Vinisth
4 - Sabe Tudo
Mensagens: 1244
Registrado em: Qui 10 Jun, 2010 23:39
Última visita: 11-07-23
Jan 2015 09 11:58

Re: Números primos

Mensagem não lida por Vinisth »

Ol,á a todos,

Primeiro faz-se o teste com p=2, como sousóeu fez.
Todo primo p>2 é ímpar, logo seja p da foma p=2n+1 \ ,\ n \in N \implies 2^{2n}-1=(2^n-1)(2^n+1)=(2n+1)x^2
Como mdc(2^n-1, 2^n+1)=1 e cada fator é um número ímpar consecutivo, a equação se torna em resolver :
\begin{cases}
2^n-1=py^2 \\ 
2^n+1=t^2
\end{cases} \ \ \ ou  \ \ \ \begin{cases}
2^n-1=t^2 \\ 
2^n+1=py^2
\end{cases}

2^n=t^2-1=(t-1)(t+1)
Ambos fatores devem ser potências de 2, isso só acontece
\iff t-1=2^u \, \ , t+1=2^v \implies 2^v-2^u=2 \implies v=2\ , u=1
Logo, só tem solução para t=3 \implies n=3 \implies p=7, analogamente pode-se ter p=3.

Os números são \{3,7\}
Para complemento: Fermat Quotient

Abraço !

Última edição: Vinisth (Sex 09 Jan, 2015 11:58). Total de 1 vez.



Responder
  • Tópicos Semelhantes
    Respostas
    Exibições
    Última msg

Voltar para “Olimpíadas”